User Avatar
ddhanya.d23
Joined
Apr 2025
Subscription
Free
User Avatar
ddhanya.d23
Saturday, May 31

im just thinking of weakening here, even though this is a strengthen question.

if it asks to weaken, would it look something like this:

"people moving into the city are ones looking to start new families and people moving out are ones that were living alone" - this would imply population might actually not decline, while also does not contradict premise "households moving out is greater than households moving in"

its not a great weakener in the sense that we dont know if pop would grow or remain the same, but hopefully not decline like the conclusion claims.

User Avatar
ddhanya.d23
Thursday, May 29

at this point i dont see any patterns in identifying the right answer choice.

i dont know if its cause the lessons are not working for me or if its because the questions are actually hard.

i chose D thinking, how does this weaken? it doesnt do anything.

I didnt chose B because i translated that as "some victims of Ebola virus are NOT afflicted with hiccups" and that aligns with the stimulus saying "many victims of ebola mention hiccups"

if it's true that SOME mention hiccups, then SOME can NOT mention hiccups too.

correct me if my last statement is a very wrong interpretation of "some"

User Avatar
ddhanya.d23
Wednesday, May 28

i would have thought D and E would weaken the argument. do they?

User Avatar
ddhanya.d23
Tuesday, May 20

i think the biggest assumption we stick to when we ruled out the wrong answer choices is that

"the percentage of people in group 2 (65 and younger) is less than the percentage of people in group 1 (65 and older) which is 25%"

the last sentence compares % of people within the same group 2, or atleast thats what i understand from the explanation.

25% is not even more than half the group 1, so it wont even be half the entire population of 5000.

say that, group 2 makes up more than half the entire population of 5000. and the % that are malnourished in group 2 is 30% and % that fall below poverty in group 2 is 55%.

the last statement will still be true.

but then makes ruling out the wrong answers confusing, because 30% of group 2 is greater than 25% of group 1.

correct me if my understanding of the last statement is wrong.

User Avatar
ddhanya.d23
Thursday, May 08

#feedback In the videos, it would be helpful to see all the answer choices drawn out in Lawgic. Helps to compare the Lawgic statements in answer choices to the one drawn from the stimulus.

Also, more drills for "you try". Great stuff as always!

User Avatar
ddhanya.d23
Wednesday, May 07

is it right to say "less than half" instead of "ranges from none to half"?

User Avatar
ddhanya.d23
Tuesday, May 06

i noticed this for Q1 :

the "embedded conditional" framework seems like it can only be formatted in one way where out of the two ideas "poachers driven out" and "pandas relocated will not prosper", the first idea need to be kept as the sufficient term. second idea is expressed as conditional. this way, the sufficient of the second idea's conditional form (pandas relocated --> /prosper) can be taken out to make the final statement: /poachers driven out AND pandas relocated --> /prosper

User Avatar
ddhanya.d23
Wednesday, Jun 04

#feedback i found it much more helpful when the mapping that starts at 3:30 was introduced AFTER we read Simpson's argument. Because the mapping is 99% related to simpson's argument just like the question stem asks.

My understanding of simpson's argument is:

fact : it is small because curators believe there is very little high quality contemporary art.

-----------------------

conclusion: small size is appropriate

then the rule that can be applied to simpson's argument is: high quality art is a should factor for a museum to be appropriate

but then the right answer choice does not have "appropriate" in the rule. which is when we consider vandenburg's argument about a museum as a whole, including it's size and purpose.

User Avatar
ddhanya.d23
Monday, Jun 02

i approached this as a sufficient assumption question because of the phrase "if assumed" in the question stem.

then A was pretty straight forward. A focuses on that assumption and does not try to weaken it. So thats good. then the "sufficient assumption" is the part where you link the premise and the conclusion as tight as possible.

did not pick D because it goes against the author's assumption.

to strengthen, you want to hold on to that assumption STRONGLY.

User Avatar
ddhanya.d23
Friday, May 02

I think i tripped on Q3 at first.

"only" belongs to group 2 Necessary indicators. So I wrote "oral myth survived" as the necessary condition.

However, in common sense, oral myths are not myths that are written down, This makes "oral myth written down" the necessary condition

"If oral myth survived, then oral myth written down eventually."

In other words, It is necessary for oral myth to have been written down for oral myth to survive. This condition makes sense.

Hoping I'm right. The venn diagram way also makes sense.

User Avatar
ddhanya.d23
Friday, May 02

i found this helpful as a way of notating, in case someone else finds it helpful too:

strip away the modifiers and you'll get:

"zombies attack, market crash"

in notation it becomes ZA --> MC

basically, remove the modifers such that the conditional logic still makes sense in English.

Then assign notations to the logic in English

User Avatar
ddhanya.d23
Friday, May 02

i love that he asked a bonus question - whether there was any assumption. as soon as i read that, i went back to the stimulus and found the assumption. nice!

Confirm action

Are you sure?